Conjetura relacionada con la ley de Kolmogorov 0-1 (para eventos)


8

Sea un espacio de probabilidad. Conjetura:(Ω,F,P)

Supongamos que tenemos eventos st , o . Existe una secuencia independiente de eventos stA1,A2,... Anσ(An,An+1,...)P(A)=01B1,B2,...

τAn:=nσ(An,An+1,...)=nσ(Bn,Bn+1,...):=τBn

¿Es esto cierto?


Creo que existe una función st 's son independientes, por lo que podemos elegir . ¿Es eso cierto? ¿Por qué por qué no? Si no, ¿de qué otra manera puedo probar o refutar la conjetura anterior? Si es cierto, creo que se puede probar modificando la prueba de la Ley Kolmogorov 0-1 (para eventos).f:NNAf(n)Bn=Af(n)


Quizás una de estas subsecuencias de conjuntos es independiente:

An

A2n,A2n+1

A3n,A3n+1,A3n+2

Amn,Amn+1,Amn+2,...,Amn+(m1)

Creo que tenemos eso

τAn=τAmn+i:=nσ(Amn+i,Am(n+1)+i,...)

donde e .mNi{0,1,2,...,m1}


Parece que necesitamos tal , si existe, para satisfacer la siguiente condición:f(n)

(**)σ(Af(n),Af(n+1)...)σ(An,An+1,...)

lo que supongo es cierto si (y solo si?)f(n)n .


Otros posibles candidatos para :f(n) (suponga que las variables son st se cumple. Si es necesario, o también).f:NN()f(n)n

  1. i=0maini

  2. 2n,3n,...

  3. i=1mbicin

  4. tn,tn ( supongo quet>e1/e )

  5. i=1mbicin,i=1mbicin

  6. linear combination of trigonometric functions,linear combination of trigonometric functions

  7. Some linear combination of the above,Some linear combination of the above


Suponiendo que la conjetura es cierta , supongo que no es necesario encontrar que funcione para todas las secuencias posibles de eventos porque tal puede que ni siquiera exista.f(n)A1,A2,...f(n)


Para refutar la conjetura : supongo que debemos demostrar que dicha secuencia siendo independiente implica que tail nunca será igual a tail ya que tail será trivial por la ley de Kolmogorov 0-1 (para eventos).BnBnAnBnP

Algo que podría ayudar: podríamos mostrar que o y no es independiente, pero no estoy seguro de que la conjetura sea refutada porque podría construir algunos que se vean así: Anσ(Af(n),Af(n+1),...),P(A)=01nN,Af(n),Af(n+1),...Bn

  1. Bn=An+1An
  2. Bn=AnAn1,A0=
  3. Bn=mAmn
  4. Bn=mAmn
  5. B2n=mAmn,B2n+1=mAmn
  6. Bn=lim supmAmn
  7. Bn=lim infmAmn
  8. B2n=lim supmAmn,B2n+1=lim infmAmn

No quiere decir, por supuesto, que ninguno de esos satisfaga pero que no necesita estar en la forma .BnτAn=τBnBnAf(n)


Borel-Cantelli:

  1. Si . Por tanto, es independiente.nP(An)<0=P(lim supAn)=P(lim supAmn) mNBm=lim supAmn

  2. Si , entonces tal vez esta extensión de Borel-Cantelli ? No estoy seguro de entenderlo o de cómo sería útil. No creo que podamos concluir nada si tenemos .nP(An)=P(lim supAn)

  3. Luego está el caso de pero las condiciones anteriores no se cumplen.nP(An)=


1
¿Quizás una prueba por construcción, donde ? B1=A1,B2=A2A1,
jbowman

1
Para mí, esta conjetura parece poco probable que sea cierta a menos que agregue condiciones adicionales, o quiera decir que las dos terminaciones de álgebra están de acuerdo (lo que es casi trivial). Sin embargo, no puedo ver un contraejemplo. σ
P.Windridge

1
En cualquier caso, creo que puede comenzar con la pregunta (más simple): "Sea un espacio de probabilidad. Suponga que es un álgebra generado de forma contable y que o para cualquier evento . ¿Hay una secuencia de eventos independientes en con cola álgebra ?(Ω,F,P)GFσP(A)=01AGB1,B2,FσG
P.Windridge

2
Un álgebra se genera contablemente si existe st . Es sencillo encontrar ejemplos en los que la cola álgebra no se genera de forma contable. σGF1,F2,G=σ(F1,F2,)σ
P.Windridge

2
De manera más general, ¡un sub- álgebra de un álgebra generado contablemente puede no generarse contablemente! Realmente mire el ejercicio 1.1.18 en math.mit.edu/~dws/175/prob01.pdfσσ
P.Windridge

Respuestas:


7

Si desea eventos que sean independientes de una manera interesante (no simplemente porque o ), entonces la conjetura es falsa.BnP(Bn)=0P(Bn)=1

Aquí hay un ejemplo pedante. Suponga que es un espacio de probabilidad adecuadamente rico. (Ω,F,P)

Deje que sea -nulo, es decir, . Tome , de modo que la cola álgebra sea .AFPP(A)=0Ai=AσG={,A,Ac,Ω}

Tenga en cuenta que en particular es finito.G

Ahora, suponga que es una secuencia independiente de eventos con delimitada por y . Entonces la cola álgebra no se genera contablemente. (Véase, por ejemplo, el ejercicio 1.1.18 http://math.mit.edu/~dws/175/prob01.pdf , que utiliza un argumento como el que describí anteriormente: cualquier -trivial -algebra generado de forma contable un átomo de masa , pero no tiene tal átomo).B1,B2,P(Bn)01σHPσ1H

Entonces, es finito pero ni siquiera se genera de manera contable.GH


Edición 2: si aceptaP(Bn)=0 , puede replicar cualquier -trivial álgebra generado de forma contable . Con más detalle, suponga que es generado por los eventos . Si es -trivial, entonces son todos independientes, en virtud de ser nulo (o es nulo). Ahora haga una construcción triangular para los eventos : , , . PσGE1,E2,GFGPEnEncBB1,1=E1B2,1=E1,B2,2=E2,,Bk,j=Ej1jk

Entonces es una secuencia contable (con ordenamiento natural para los índices) de eventos independientes cuya cola álgebra es .(Bk,j)σG

Entonces, aquí creo que es la pregunta clave: supongamos que es un -trivial tail -algebra generado de manera no contable (proveniente de eventos no nulos que podrían ser dependientes). ¿Se puede realizar como la cola álgebra para algunos eventos nulos?GPσGσ

Edición 1: un área gris es lo que sucede si acepta , aunque ese no parece ser el objetivo de la pregunta original.P(Bn)0


Gracias P.Windridge, pero no estoy seguro de entenderlo. 1 Si incluimos o , ¿la conjetura es (trivialmente?) Verdadera? 2 ¿Es lo que estás tratando de probar en Edit 2? Si es así, ¿es su igual a mi ? Edité OP para taquigrafíaP(Bn)=01GτAn
BCLC

Yo leo el ejercicio. ? H=τBn
BCLC

Hola BCLC, (1) Estoy diciendo que si incluimos entonces la conjetura es verdadera para todas las elecciones de los eventos que tienen una "bonita" cola álgebra (donde "agradable" aquí significa contable generado). (2) Sí y es su . Nota: el ejercicio vinculado usa " " para lo que debería ser tu , y " " denota una secuencia candidata de eventos generadores (utilizada para obtener una contradicción.P(Bn)=0A1,A2,σG=τ(An)Hτ(Bn)AnBnBn
P.Windridge

No estoy seguro de seguirlo. Solo por los supuestos, ¿los SON independientes? Ai
BCLC

En el ejercicio 1.1.18 de math.mit.edu/~dws/175/prob01.pdf , los son eventos independientes, que debe considerar como los en su conjetura. ¿Es eso lo que preguntabas? AiBi
P.Windridge el
Al usar nuestro sitio, usted reconoce que ha leído y comprende nuestra Política de Cookies y Política de Privacidad.
Licensed under cc by-sa 3.0 with attribution required.